Variable aléatoire uniforme comme somme de deux variables aléatoires


18

Extrait de Grimmet et Stirzaker :

Montrer qu'il ne peut pas être le cas où U=X+YU est uniformément distribué sur [0,1] et et sont indépendants et identiques. Vous ne devez pas supposer que X et Y sont des variables continues.XY

Une simple preuve de contradiction suffit pour le cas où , sont supposés discrets en arguant qu'il est toujours possible de trouver un et tels que tandis que .XYuuP(Uu+u)P(Uu)P(X+Yu)=P(X+Yu+u)

Cependant cette preuve ne s'étend pas à étant absolument continu ou singulier continu. Conseils / Commentaires / Critique?X,Y


3
Astuce : les fonctions caractéristiques sont vos amis.
Cardinal

1
X et Y sont iid donc leurs fonctions caractéristiques doivent être identiques. Cependant, vous devez utiliser la fonction caractéristique et non la fonction de génération de moment - le mgf n'est pas garanti pour X, donc montrer que le mgf a une propriété impossible ne signifie pas qu'il n'y a pas un tel X. Tous les VR ont une fonction caractéristique, donc si vous montrez qu'il a une propriété impossible, alors il n'y a pas de tel X.
Silverfish

1
Si les distributions de X et Y ont des atomes , disons que P{X=a}=P{Y=a}=b>0 , alors P{X+Y=2a}b2>0 et donc X+Y ne peut pas être uniformément distribué sur [0,1]. Ainsi, il n'est pas nécessaire de considérer le cas des distributions de X et Y ayant des atomes.
Dilip Sarwate

Réponses:


13

Le résultat peut être prouvé par une image: les zones grises visibles montrent qu'une distribution uniforme ne peut pas être décomposée comme une somme de deux variables indépendantes identiquement distribuées.

Notation

Soit X et Y iid tels que X+Y ait une distribution uniforme sur [0,1] . Cela signifie que pour tout 0ab1 ,

Pr(a<X+Yb)=ba.

Le support essentiel de la distribution commune des X et Y est donc [0,1/2] (car autrement il y aurait une probabilité positive que X+Y se situe en dehors de [0,1] ).

L'image

Laissez 0<ϵ<1/4 . Contemplez ce diagramme montrant comment les sommes des variables aléatoires sont calculées:

Figure

La distribution de probabilité sous-jacente est celle conjointe pour . La probabilité de tout événement a < X + Y b est donnée par la probabilité totale couverte par la bande diagonale s'étendant entre les lignes x + y = a et x + y = b . Trois de ces bandes sont représentées: de 0 à ϵ , apparaissant comme un petit triangle bleu en bas à gauche; à partir de 1 / deux - ε de 1 / deux(X,Y)a<X+Ybx+y=ax+y=b0ϵ1/2ϵ , représenté par un rectangle gris coiffé de deux triangles (jaune et vert); et de 1 - ϵ à 1 , apparaissant comme un petit triangle rouge en haut à droite.1/2+ϵ1ϵ1

Ce que l'image montre

En comparant le triangle inférieur gauche de la figure au carré inférieur gauche qui le contient et en exploitant l'hypothèse iid pour et Y , il est clair queXY

ϵ=Pr(X+Yϵ)<Pr(Xϵ)Pr(Yϵ)=Pr(Xϵ)2.

Notez que l'inégalité est stricte: l'égalité n'est pas possible car il existe une probabilité positive que et Y soient inférieurs à ϵ mais néanmoins X + Y > ϵ .XYϵX+Y>ϵ

De même, en comparant le triangle rouge au carré dans le coin supérieur droit,

ϵ=Pr(X+Y>1ϵ)<Pr(X>1/2ϵ)2.

Enfin, la comparaison des deux triangles opposés en haut à gauche et en bas à droite avec la bande diagonale les contenant donne une autre inégalité stricte,

2ϵ<2Pr(Xϵ)Pr(X>1/2ϵ)<Pr(1/2ϵ<X+Y1/2+ϵ)=2ϵ.

Les premiers ensuit d'inégalité des deux précédents (prennent leurs racines carrées et les multiplier) tandis que le second décrit le (strict) l' inclusion des triangles dans la bande et la dernière égalité exprime l'uniformité de . La conclusion que 2 ϵ < 2 ϵ est la contradiction prouvant que de tels X et Y ne peuvent pas exister, QED .X+Y2ϵ<2ϵXY


3
(+1) J'aime cette approche. En récupérant mon dos d'une enveloppe dans la corbeille à papier, je peux voir que j'ai dessiné le même schéma, sauf que je n'ai pas marqué sur les triangles jaunes et verts à l'intérieur de la bande. J'ai obtenu les inégalités pour les triangles bleu et rouge. J'ai joué avec eux et quelques autres probabilités, mais je n'ai jamais pensé à enquêter sur la probabilité de la bande, ce qui s'avère être l'étape critique. Je me demande quel processus de réflexion a pu motiver cette idée?
Silverfish

En fait, où @whuber a des triangles jaunes et verts, j'ai dessiné sur des carrés (j'avais effectivement décomposé dans une grille). En regardant l'étape qui "décrit le (strict) inclusion des triangles dans la bande", 2 Pr ( X ε ) Pr ( X > 1 / 2 - ε ) < Pr ( 1 / 2 - ε < X + Y une / 2 + ε )[0,0.5]22Pr(Xϵ)Pr(X>1/2ϵ)<Pr(1/2ϵ<X+Y1/2+ϵ), Je me demande si ce serait réellement plus géométriquement naturel avec des carrés coiffant la bande que des triangles?
Silverfish

1
@Silver m'a rappelé une analyse des sommes de distributions uniformes que j'ai publiées il y a quelques années. Cela a suggéré de visualiser la somme géométriquement. Il était immédiatement évident que beaucoup de probabilité devait être concentrée près des coins ( 0 , 0 ) et ( 1 / 2 , 1 / 2 ) afin de la somme soit uniforme et probabilité relativement peu d'être près de la diagonale du centre X + Y = une / 2 . Cela a conduit au diagramme que j'ai redessiné dans Mathematica.X+Y(0,0)(1/2,1/2)X+Y=1/2À ce stade, la réponse s'est écrite. Oui, l'utilisation de carrés dans la bande centrale peut être plus nette.
whuber

Merci! « Notez que l'inégalité est stricte: l' égalité est pas possible parce qu'il ya une probabilité positive que soit de ou Y est inférieur à ε mais néanmoins X + Y > ε . » Je ne suis pas sûr de suivre cela. Il me semble que le but ici est de montrer Pr ( X + Y e ) < Pr ( X e Y e ) , ne pas exiger une probabilité positive pour un événement A dans lequel les deux X etXYϵX+Y>ϵPr(X+Yϵ)<Pr(XϵYϵ)A X est inférieur ou égal à ϵ et pourtant X + Y > ϵ ? C'est le "l'un des" vs "les deux" je hésite. YϵX+Y>ϵ
Silverfish

@Silverfish Merci; Je n'ai pas exprimé cela comme je l'avais prévu. Vous avez raison: le langage vise essentiellement à décrire la portion d'un petit carré qui n'est pas à l'intérieur du triangle.
whuber

10

J'ai essayé de trouver une preuve sans considérer les fonctions caractéristiques. Un excès de kurtosis fait l'affaire. Voici la réponse sur deux lignes: puisque X et Y sont iid. Alors Kurt ( U ) = - 1.2 implique Kurt ( X ) = - 2.4 qui est une contradiction comme Kurt ( X )Kurt(U)=Kurt(X+Y)=Kurt(X)/2XYKurt(U)=1.2Kurt(X)=2.4 pour toute variable aléatoire.Kurt(X)2

La ligne de raisonnement qui m'a amené à ce point est plus intéressante. (et Y ) doit être borné entre 0 et 0,5 - c'est évident, mais cela signifie que ses moments et ses moments centraux existent. Commençons par considérer la moyenne et la variance: E ( U ) = 0,5 et Var ( U ) = 1XYE(U)=0.5 . SiXetYsont distribués de manière identique, nous avons:Var(U)=112XY

E(X+Y)=E(X)+E(Y)=2E(X)=0.5

Donc . Pour la variance, nous devons également utiliser l'indépendance pour appliquer:E(X)=0.25

Var(X+Y)=Var(X)+Var(Y)=2Var(X)=112

D'où etσX=1Var(X)=124. Hou la la! C'est beaucoup de variation pour une variable aléatoire dont le support va de 0 à 0,5. Mais nous aurions dû nous en douter, puisque l'écart-type ne va pas évoluer de la même manière que la moyenne.σX=1260.204

Maintenant, quel est le plus grand écart-type qu'une variable aléatoire peut avoir si la plus petite valeur qu'elle peut prendre est 0, la plus grande valeur qu'elle peut prendre est 0,5 et la moyenne est 0,25? La collecte de toute la probabilité à deux masses ponctuelles sur les extrêmes, à 0,25 de la moyenne, donnerait clairement un écart-type de 0,25. Notre est donc grand mais pas impossible. (J'espérais montrer que cela impliquait trop de probabilité dans les queues pour que X + Y soit uniforme, mais je ne pouvais aller nulle part avec ça au dos d'une enveloppe.)σXX+Y

Les considérations du deuxième moment imposent presque une contrainte impossible à alors considérons les moments supérieurs. Qu'en est-il du coefficient d'inclinaison du moment de Pearson , γ 1 = E ( X - μ X ) 3X ? Cela existe puisque les moments centraux existent etσX0. Il est utile de connaître certaines propriétés des cumulants, en particulier l'application d'une indépendance puis d'une distribution identique donne:γ1=E(XμX)3σX3=κ3κ23/2σX0

κi(U)=κi(X+Y)=κi(X)+κi(Y)=2κi(X)

This additivity property is precisely the generalisation of how we dealt with the mean and variance above - indeed, the first and second cumulants are just κ1=μ and κ2=σ2.

Then κ3(U)=2κ3(X) and (κ2(U))3/2=(2κ2(X))3/2=23/2(κ2(X))3/2. The fraction for γ1 cancels to yield Skew(U)=Skew(X+Y)=Skew(X)/2. Since the uniform distribution has zero skewness, so does X, but I can't see how a contradiction arises from this restriction.

So instead, let's try the excess kurtosis, γ2=κ4κ22=E(XμX)4σX43. By a similar argument (this question is self-study, so try it!), we can show this exists and obeys:

Kurt(U)=Kurt(X+Y)=Kurt(X)/2

The uniform distribution has excess kurtosis 1.2 so we require X to have excess kurtosis 2.4. But the smallest possible excess kurtosis is 2, which is achieved by the Binomial(1,12) Bernoulli distribution.


2
(+1) This is a quite clever approach, which was new to me. Thanks. Note that some of your analysis could have been streamlined by considering a uniform centered at zero. (The equivalence of the problem is immediate.) That would have immediately told you that considering skew was a dead-end.
cardinal

@cardinal: I knew the skew was a dead-end before I worked on it. The purpose was expository: it's a self-study question so I didn't want to solve it in full! Rather I wanted to leave a hint on how to deal with the next level up...
Silverfish

@cardinal: I was in two minds whether to center or not. I did back-of-envelope calculations more conveniently, but in the final analysis we just need (1) a simple case of the general result that Kurt(X1+...+Xn)=1nKurt(X) for iid Xi, (2) that Kurt(U)=1.2 for any uniform distribution, and (3) Kurt(X) exists since X is bounded and σX0 (which is trivial, else σU=0). So none of the key results actually required centering, though bits may have looked less ugly!
Silverfish

Yes, the word "streamlined" was carefully chosen. :-) I did not intend my comment to be read as criticism of your exposition. Cheers.
cardinal

@cardinal Incidentally, variance considerations alone almost worked, but the uniform isn't quite spread out enough. With a bit more probability mass nearer the extremes, e.g. fT(t)=12t2 on [-0.5, 0.5], then Var(T)=.15 and if T=X1+X2 then σX=.15/20.27>0.25 which is impossible as X is bounded by -0.25 and 0.25. Of course, you will see immediately how this relates to the present example! I wonder if the approach generalises, I'm sure other bounded RVs can't be decomposed into sums but require even higher moments investigated to find the contradiction.
Silverfish
En utilisant notre site, vous reconnaissez avoir lu et compris notre politique liée aux cookies et notre politique de confidentialité.
Licensed under cc by-sa 3.0 with attribution required.